Finding the sum of squares of the real roots

The name of the pictureThe name of the pictureThe name of the pictureClash Royale CLAN TAG#URR8PPP











up vote
6
down vote

favorite
1













Let $r_1,r_2,r_3,cdots,r_n$ be the distinct real zeroes of the equation $$x^8-14x^4-8x^3-x^2+1=0.$$ Then $r_1^2+r_2^2+r_3^2+cdots+r_n^2$ is $$(A),3quad(B),14quad(C),8quad(D),16$$




I can get the sum of the squares of all roots using Vieta’s formulae, but I don't know actually how to proceed in this question.



Do I need to draw a graph and then find the answers or is there a sum trick here which I am not able to see through?










share|cite|improve this question























  • Once you get Vieta's formulae you can continue with Newton's identity, basically $a_ns_2+a_n-1s_1+a_n-2=0$. But as dxiv said, you have first to isolate the polynomial with real roots.
    – zwim
    34 mins ago














up vote
6
down vote

favorite
1













Let $r_1,r_2,r_3,cdots,r_n$ be the distinct real zeroes of the equation $$x^8-14x^4-8x^3-x^2+1=0.$$ Then $r_1^2+r_2^2+r_3^2+cdots+r_n^2$ is $$(A),3quad(B),14quad(C),8quad(D),16$$




I can get the sum of the squares of all roots using Vieta’s formulae, but I don't know actually how to proceed in this question.



Do I need to draw a graph and then find the answers or is there a sum trick here which I am not able to see through?










share|cite|improve this question























  • Once you get Vieta's formulae you can continue with Newton's identity, basically $a_ns_2+a_n-1s_1+a_n-2=0$. But as dxiv said, you have first to isolate the polynomial with real roots.
    – zwim
    34 mins ago












up vote
6
down vote

favorite
1









up vote
6
down vote

favorite
1






1






Let $r_1,r_2,r_3,cdots,r_n$ be the distinct real zeroes of the equation $$x^8-14x^4-8x^3-x^2+1=0.$$ Then $r_1^2+r_2^2+r_3^2+cdots+r_n^2$ is $$(A),3quad(B),14quad(C),8quad(D),16$$




I can get the sum of the squares of all roots using Vieta’s formulae, but I don't know actually how to proceed in this question.



Do I need to draw a graph and then find the answers or is there a sum trick here which I am not able to see through?










share|cite|improve this question
















Let $r_1,r_2,r_3,cdots,r_n$ be the distinct real zeroes of the equation $$x^8-14x^4-8x^3-x^2+1=0.$$ Then $r_1^2+r_2^2+r_3^2+cdots+r_n^2$ is $$(A),3quad(B),14quad(C),8quad(D),16$$




I can get the sum of the squares of all roots using Vieta’s formulae, but I don't know actually how to proceed in this question.



Do I need to draw a graph and then find the answers or is there a sum trick here which I am not able to see through?







polynomials roots






share|cite|improve this question















share|cite|improve this question













share|cite|improve this question




share|cite|improve this question








edited 18 mins ago









egreg

167k1180189




167k1180189










asked 1 hour ago









ashish deo singh

1485




1485











  • Once you get Vieta's formulae you can continue with Newton's identity, basically $a_ns_2+a_n-1s_1+a_n-2=0$. But as dxiv said, you have first to isolate the polynomial with real roots.
    – zwim
    34 mins ago
















  • Once you get Vieta's formulae you can continue with Newton's identity, basically $a_ns_2+a_n-1s_1+a_n-2=0$. But as dxiv said, you have first to isolate the polynomial with real roots.
    – zwim
    34 mins ago















Once you get Vieta's formulae you can continue with Newton's identity, basically $a_ns_2+a_n-1s_1+a_n-2=0$. But as dxiv said, you have first to isolate the polynomial with real roots.
– zwim
34 mins ago




Once you get Vieta's formulae you can continue with Newton's identity, basically $a_ns_2+a_n-1s_1+a_n-2=0$. But as dxiv said, you have first to isolate the polynomial with real roots.
– zwim
34 mins ago










3 Answers
3






active

oldest

votes

















up vote
4
down vote













Hint:   the real roots are the roots of the first factor:



$$
beginalign
x^8colorred+2x^4-2x^4-14x^4-8x^3-x^2+1 &= (x^4+1)^2 - x^2(4x+1)^2 \
&= (x^4 - 4 x^2 - x+1)(x^4+4x^2+x+1)
endalign
$$






share|cite|improve this answer



























    up vote
    3
    down vote













    Let $$f(x)=x^8-14x^4-8x^3-x^2+1implies f'(x)=8x^7-56x^3-24x^2-2x=0$$ for critical points. Then clearly $x=0$ is one, and is a maximum since $f''(0)<0$; $f(0)=1$.



    We get $$g(x)=4x^6-28x^3-12x-1=0$$ and since $g$ is continuous, by the Mean Value Theorem, there is a minimum between $(-0.2,0)$ and a maximum between $(-0.4,-0.2)$. At these $x$, $f(x)$ is above the $x$-axis.



    Hence there are at most four roots left. In a similar way, we test at $0.2$ intervals, and we find that $$f(-1.8)>0,quad f(-1.6)<0\f(-0.8)<0,quad f(-0.6)>0\f(0.2)>0,quad f(0.4)<0\f(2)<0,quad f(2.2)>0.$$



    Thus we have that $$(-1.6)^2+(-0.2)^2+0.2^2+2^2<r_1^2+r_2^2+r_3^2+r_4^2<(-1.8)^2+(-0.8)^2+0.4^2+2.2^2$$ or that $$6.64<r_1^2+r_2^2+r_3^2+r_4^2<8.88$$ so the only option must be $(C)$.






    share|cite|improve this answer




















    • but this is too much of approximation and working through options. wont there be any other way to find exact ans
      – ashish deo singh
      1 hour ago










    • Yes, @dxiv has given a much neater one.
      – TheSimpliFire
      1 hour ago

















    up vote
    0
    down vote













    Following the foundamental hint by dxiv, for $x^4-4x^2-x+1$ by Vieta's formula we have



    • $S_1=sum r_i=-a_3=0$

    • $S_2=sum r_ir_j=a_2=-4$

    • $S_3=sum r_ir_jr_k=-a_1=1$

    • $S_4=r_1r_2r_3r_4=a_0=1$

    and by Newton's sums we have that



    • $P_1=sum r_i=S_1=0$

    • $P_2=sum r_i^2=S_1P_1-2S_2=8$





    share|cite|improve this answer




















      Your Answer




      StackExchange.ifUsing("editor", function ()
      return StackExchange.using("mathjaxEditing", function ()
      StackExchange.MarkdownEditor.creationCallbacks.add(function (editor, postfix)
      StackExchange.mathjaxEditing.prepareWmdForMathJax(editor, postfix, [["$", "$"], ["\\(","\\)"]]);
      );
      );
      , "mathjax-editing");

      StackExchange.ready(function()
      var channelOptions =
      tags: "".split(" "),
      id: "69"
      ;
      initTagRenderer("".split(" "), "".split(" "), channelOptions);

      StackExchange.using("externalEditor", function()
      // Have to fire editor after snippets, if snippets enabled
      if (StackExchange.settings.snippets.snippetsEnabled)
      StackExchange.using("snippets", function()
      createEditor();
      );

      else
      createEditor();

      );

      function createEditor()
      StackExchange.prepareEditor(
      heartbeatType: 'answer',
      convertImagesToLinks: true,
      noModals: false,
      showLowRepImageUploadWarning: true,
      reputationToPostImages: 10,
      bindNavPrevention: true,
      postfix: "",
      noCode: true, onDemand: true,
      discardSelector: ".discard-answer"
      ,immediatelyShowMarkdownHelp:true
      );



      );













       

      draft saved


      draft discarded


















      StackExchange.ready(
      function ()
      StackExchange.openid.initPostLogin('.new-post-login', 'https%3a%2f%2fmath.stackexchange.com%2fquestions%2f2920549%2ffinding-the-sum-of-squares-of-the-real-roots%23new-answer', 'question_page');

      );

      Post as a guest






























      3 Answers
      3






      active

      oldest

      votes








      3 Answers
      3






      active

      oldest

      votes









      active

      oldest

      votes






      active

      oldest

      votes








      up vote
      4
      down vote













      Hint:   the real roots are the roots of the first factor:



      $$
      beginalign
      x^8colorred+2x^4-2x^4-14x^4-8x^3-x^2+1 &= (x^4+1)^2 - x^2(4x+1)^2 \
      &= (x^4 - 4 x^2 - x+1)(x^4+4x^2+x+1)
      endalign
      $$






      share|cite|improve this answer
























        up vote
        4
        down vote













        Hint:   the real roots are the roots of the first factor:



        $$
        beginalign
        x^8colorred+2x^4-2x^4-14x^4-8x^3-x^2+1 &= (x^4+1)^2 - x^2(4x+1)^2 \
        &= (x^4 - 4 x^2 - x+1)(x^4+4x^2+x+1)
        endalign
        $$






        share|cite|improve this answer






















          up vote
          4
          down vote










          up vote
          4
          down vote









          Hint:   the real roots are the roots of the first factor:



          $$
          beginalign
          x^8colorred+2x^4-2x^4-14x^4-8x^3-x^2+1 &= (x^4+1)^2 - x^2(4x+1)^2 \
          &= (x^4 - 4 x^2 - x+1)(x^4+4x^2+x+1)
          endalign
          $$






          share|cite|improve this answer












          Hint:   the real roots are the roots of the first factor:



          $$
          beginalign
          x^8colorred+2x^4-2x^4-14x^4-8x^3-x^2+1 &= (x^4+1)^2 - x^2(4x+1)^2 \
          &= (x^4 - 4 x^2 - x+1)(x^4+4x^2+x+1)
          endalign
          $$







          share|cite|improve this answer












          share|cite|improve this answer



          share|cite|improve this answer










          answered 1 hour ago









          dxiv

          56.1k64798




          56.1k64798




















              up vote
              3
              down vote













              Let $$f(x)=x^8-14x^4-8x^3-x^2+1implies f'(x)=8x^7-56x^3-24x^2-2x=0$$ for critical points. Then clearly $x=0$ is one, and is a maximum since $f''(0)<0$; $f(0)=1$.



              We get $$g(x)=4x^6-28x^3-12x-1=0$$ and since $g$ is continuous, by the Mean Value Theorem, there is a minimum between $(-0.2,0)$ and a maximum between $(-0.4,-0.2)$. At these $x$, $f(x)$ is above the $x$-axis.



              Hence there are at most four roots left. In a similar way, we test at $0.2$ intervals, and we find that $$f(-1.8)>0,quad f(-1.6)<0\f(-0.8)<0,quad f(-0.6)>0\f(0.2)>0,quad f(0.4)<0\f(2)<0,quad f(2.2)>0.$$



              Thus we have that $$(-1.6)^2+(-0.2)^2+0.2^2+2^2<r_1^2+r_2^2+r_3^2+r_4^2<(-1.8)^2+(-0.8)^2+0.4^2+2.2^2$$ or that $$6.64<r_1^2+r_2^2+r_3^2+r_4^2<8.88$$ so the only option must be $(C)$.






              share|cite|improve this answer




















              • but this is too much of approximation and working through options. wont there be any other way to find exact ans
                – ashish deo singh
                1 hour ago










              • Yes, @dxiv has given a much neater one.
                – TheSimpliFire
                1 hour ago














              up vote
              3
              down vote













              Let $$f(x)=x^8-14x^4-8x^3-x^2+1implies f'(x)=8x^7-56x^3-24x^2-2x=0$$ for critical points. Then clearly $x=0$ is one, and is a maximum since $f''(0)<0$; $f(0)=1$.



              We get $$g(x)=4x^6-28x^3-12x-1=0$$ and since $g$ is continuous, by the Mean Value Theorem, there is a minimum between $(-0.2,0)$ and a maximum between $(-0.4,-0.2)$. At these $x$, $f(x)$ is above the $x$-axis.



              Hence there are at most four roots left. In a similar way, we test at $0.2$ intervals, and we find that $$f(-1.8)>0,quad f(-1.6)<0\f(-0.8)<0,quad f(-0.6)>0\f(0.2)>0,quad f(0.4)<0\f(2)<0,quad f(2.2)>0.$$



              Thus we have that $$(-1.6)^2+(-0.2)^2+0.2^2+2^2<r_1^2+r_2^2+r_3^2+r_4^2<(-1.8)^2+(-0.8)^2+0.4^2+2.2^2$$ or that $$6.64<r_1^2+r_2^2+r_3^2+r_4^2<8.88$$ so the only option must be $(C)$.






              share|cite|improve this answer




















              • but this is too much of approximation and working through options. wont there be any other way to find exact ans
                – ashish deo singh
                1 hour ago










              • Yes, @dxiv has given a much neater one.
                – TheSimpliFire
                1 hour ago












              up vote
              3
              down vote










              up vote
              3
              down vote









              Let $$f(x)=x^8-14x^4-8x^3-x^2+1implies f'(x)=8x^7-56x^3-24x^2-2x=0$$ for critical points. Then clearly $x=0$ is one, and is a maximum since $f''(0)<0$; $f(0)=1$.



              We get $$g(x)=4x^6-28x^3-12x-1=0$$ and since $g$ is continuous, by the Mean Value Theorem, there is a minimum between $(-0.2,0)$ and a maximum between $(-0.4,-0.2)$. At these $x$, $f(x)$ is above the $x$-axis.



              Hence there are at most four roots left. In a similar way, we test at $0.2$ intervals, and we find that $$f(-1.8)>0,quad f(-1.6)<0\f(-0.8)<0,quad f(-0.6)>0\f(0.2)>0,quad f(0.4)<0\f(2)<0,quad f(2.2)>0.$$



              Thus we have that $$(-1.6)^2+(-0.2)^2+0.2^2+2^2<r_1^2+r_2^2+r_3^2+r_4^2<(-1.8)^2+(-0.8)^2+0.4^2+2.2^2$$ or that $$6.64<r_1^2+r_2^2+r_3^2+r_4^2<8.88$$ so the only option must be $(C)$.






              share|cite|improve this answer












              Let $$f(x)=x^8-14x^4-8x^3-x^2+1implies f'(x)=8x^7-56x^3-24x^2-2x=0$$ for critical points. Then clearly $x=0$ is one, and is a maximum since $f''(0)<0$; $f(0)=1$.



              We get $$g(x)=4x^6-28x^3-12x-1=0$$ and since $g$ is continuous, by the Mean Value Theorem, there is a minimum between $(-0.2,0)$ and a maximum between $(-0.4,-0.2)$. At these $x$, $f(x)$ is above the $x$-axis.



              Hence there are at most four roots left. In a similar way, we test at $0.2$ intervals, and we find that $$f(-1.8)>0,quad f(-1.6)<0\f(-0.8)<0,quad f(-0.6)>0\f(0.2)>0,quad f(0.4)<0\f(2)<0,quad f(2.2)>0.$$



              Thus we have that $$(-1.6)^2+(-0.2)^2+0.2^2+2^2<r_1^2+r_2^2+r_3^2+r_4^2<(-1.8)^2+(-0.8)^2+0.4^2+2.2^2$$ or that $$6.64<r_1^2+r_2^2+r_3^2+r_4^2<8.88$$ so the only option must be $(C)$.







              share|cite|improve this answer












              share|cite|improve this answer



              share|cite|improve this answer










              answered 1 hour ago









              TheSimpliFire

              10.9k62154




              10.9k62154











              • but this is too much of approximation and working through options. wont there be any other way to find exact ans
                – ashish deo singh
                1 hour ago










              • Yes, @dxiv has given a much neater one.
                – TheSimpliFire
                1 hour ago
















              • but this is too much of approximation and working through options. wont there be any other way to find exact ans
                – ashish deo singh
                1 hour ago










              • Yes, @dxiv has given a much neater one.
                – TheSimpliFire
                1 hour ago















              but this is too much of approximation and working through options. wont there be any other way to find exact ans
              – ashish deo singh
              1 hour ago




              but this is too much of approximation and working through options. wont there be any other way to find exact ans
              – ashish deo singh
              1 hour ago












              Yes, @dxiv has given a much neater one.
              – TheSimpliFire
              1 hour ago




              Yes, @dxiv has given a much neater one.
              – TheSimpliFire
              1 hour ago










              up vote
              0
              down vote













              Following the foundamental hint by dxiv, for $x^4-4x^2-x+1$ by Vieta's formula we have



              • $S_1=sum r_i=-a_3=0$

              • $S_2=sum r_ir_j=a_2=-4$

              • $S_3=sum r_ir_jr_k=-a_1=1$

              • $S_4=r_1r_2r_3r_4=a_0=1$

              and by Newton's sums we have that



              • $P_1=sum r_i=S_1=0$

              • $P_2=sum r_i^2=S_1P_1-2S_2=8$





              share|cite|improve this answer
























                up vote
                0
                down vote













                Following the foundamental hint by dxiv, for $x^4-4x^2-x+1$ by Vieta's formula we have



                • $S_1=sum r_i=-a_3=0$

                • $S_2=sum r_ir_j=a_2=-4$

                • $S_3=sum r_ir_jr_k=-a_1=1$

                • $S_4=r_1r_2r_3r_4=a_0=1$

                and by Newton's sums we have that



                • $P_1=sum r_i=S_1=0$

                • $P_2=sum r_i^2=S_1P_1-2S_2=8$





                share|cite|improve this answer






















                  up vote
                  0
                  down vote










                  up vote
                  0
                  down vote









                  Following the foundamental hint by dxiv, for $x^4-4x^2-x+1$ by Vieta's formula we have



                  • $S_1=sum r_i=-a_3=0$

                  • $S_2=sum r_ir_j=a_2=-4$

                  • $S_3=sum r_ir_jr_k=-a_1=1$

                  • $S_4=r_1r_2r_3r_4=a_0=1$

                  and by Newton's sums we have that



                  • $P_1=sum r_i=S_1=0$

                  • $P_2=sum r_i^2=S_1P_1-2S_2=8$





                  share|cite|improve this answer












                  Following the foundamental hint by dxiv, for $x^4-4x^2-x+1$ by Vieta's formula we have



                  • $S_1=sum r_i=-a_3=0$

                  • $S_2=sum r_ir_j=a_2=-4$

                  • $S_3=sum r_ir_jr_k=-a_1=1$

                  • $S_4=r_1r_2r_3r_4=a_0=1$

                  and by Newton's sums we have that



                  • $P_1=sum r_i=S_1=0$

                  • $P_2=sum r_i^2=S_1P_1-2S_2=8$






                  share|cite|improve this answer












                  share|cite|improve this answer



                  share|cite|improve this answer










                  answered 23 mins ago









                  gimusi

                  72.7k73888




                  72.7k73888



























                       

                      draft saved


                      draft discarded















































                       


                      draft saved


                      draft discarded














                      StackExchange.ready(
                      function ()
                      StackExchange.openid.initPostLogin('.new-post-login', 'https%3a%2f%2fmath.stackexchange.com%2fquestions%2f2920549%2ffinding-the-sum-of-squares-of-the-real-roots%23new-answer', 'question_page');

                      );

                      Post as a guest













































































                      Comments

                      Popular posts from this blog

                      Long meetings (6-7 hours a day): Being “babysat” by supervisor

                      Is the Concept of Multiple Fantasy Races Scientifically Flawed? [closed]

                      Confectionery